Тёмный
Reach The Stars
Reach The Stars
Reach The Stars
Подписаться
Комментарии
5 дней назад
Welcome to Flat Earth.
@Hussain-px3fc
@Hussain-px3fc 7 дней назад
Aren’t A,B,C,O concyclic here? 1:58
@Hussain-px3fc
@Hussain-px3fc 8 дней назад
1:34 it can also be linked to similar triangles, great video as always 👍
@PedroHenrique-xg8no
@PedroHenrique-xg8no 11 дней назад
Could you provide the complete question, I mean the statement?
@Hussain-px3fc
@Hussain-px3fc 11 дней назад
Awesome solution. Just to show variety you can do it as well by connecting PC to get a diagonal of the square and then get it’s length by using this AP^2+PC^2=BP^2+PD^2 (can be proven by Pythagoras) and get PC = sqrt(3) then get the side of square and apply cosine rule on triangle APB
@Hussain-px3fc
@Hussain-px3fc 11 дней назад
Thank you for the clear explanation and intreating problem
@michaelg3490
@michaelg3490 20 дней назад
I think you missed a condition that a must be an integer, right?
@yakupbuyankara5903
@yakupbuyankara5903 26 дней назад
a=24.
@Quest3669
@Quest3669 28 дней назад
Replace x^2with x-1 to get instant soln
@RONEXX-263
@RONEXX-263 Месяц назад
Follow up the following link if you want to solve grade 11 or o-level mathematics like a genius ru-vid.com/video/%D0%B2%D0%B8%D0%B4%D0%B5%D0%BE-YAM-A3vAXhk.htmlsi=p_qOxs7SaHiezejV
@co2-cx6qy
@co2-cx6qy Месяц назад
Thanks for this video.But I want to ask something.q 's minimum value is not 16.q can be equal to 11.For exactly p=6 and q=11 5/11 < 6/11 < 4/7 If you look to this unequation , it is True. So why we don't take 11? Also I want to explain this solution: Please tell me if it is True or false. Firstly if these numbers are positive , we can multiply everywhere with 7q.So we will get 35<7p<4q , so p can be 6.And q can be 11 for the minimum value. (If these p,q are positive intigers.)
@reachthestars98
@reachthestars98 Месяц назад
Hi, thanks for the comment. I double checked the scenario when p=6 and q=11, and it does not satisfy 5/9<p/q<4/7. 5/9 is not less than p/q in this case. I think you read the question wrong when u said 5/11<6/11<4/7. It is supposed to be 5/9<6/11<4/7 which is not true.
@co2-cx6qy
@co2-cx6qy Месяц назад
@@reachthestars98 Thanks.I read the number 9 as q . Thanks for help.
@AshkanMustafa
@AshkanMustafa Месяц назад
Great video! Though, how do you prove that y is in the interval from -1 to 1 (y z (-1;1))
@reachthestars98
@reachthestars98 Месяц назад
To find the range of y, we can analyze the behavior of the function y=3^x. When x increases, the value of y gets very large, thus 2/(3^x+1) approaches 0, which makes y get very close to 1. When x decreases, the value of y gets very small, thus 2/(3^x+1) approaches 2, which makes y get very close to -1. So the interval would be (-1,1).
@xualain3129
@xualain3129 Месяц назад
Let <QCB=a=<PBA and side AB=BC=x x*sin a=6 (6+7)*cos a=x ->13*cos a*sin a=6 ->sin 2a=12/13 ->cos 2a=5/13 ->1-2*sin(a)^2=5/13 -> sin a=2/sqrt(13) x=6/sin a=3*sqrt(13) -> area(ABCD)=x*x=117
@reachthestars98
@reachthestars98 Месяц назад
Yes, that also works.
@harryedwardlawrence598
@harryedwardlawrence598 Месяц назад
More easy with trigonometry!!!! Tg(DPC) = infinity = tg (CBA+DAB)
@ankush_alonso
@ankush_alonso Месяц назад
Bro this is Olympiad question I solved it in less than 5 seconds by seeing the thumbnail, wtf dawn easy, if this is Olympiad level than u will cry after seeing JEE advanced maths papers
@tubahguclu
@tubahguclu Месяц назад
471
@vittoriomarcocci2159
@vittoriomarcocci2159 Месяц назад
because the three sides of dec triangle are a pithagoric tern
@sergeibodkin4477
@sergeibodkin4477 Месяц назад
Great video, thanks. Can you explain why DEC angle is a right angle, thanks?
@Ytanul
@Ytanul Месяц назад
the reciprocal of the pythagorian theoreme
@reachthestars98
@reachthestars98 Месяц назад
The Triangle satisfies the Pythagorean theorem, therefore it is a right triangle.
@tylerdurden9614
@tylerdurden9614 29 дней назад
5-12-13 triangle belongs to a right triangle multiply each by 3, so that makes a 15-36-39 triangle that also belongs to a right triangle
@yusufsenyuz
@yusufsenyuz Месяц назад
I am curious that whether the following solution is acceptable or not. Just give a number for the value of the function such as 1 for positive 1 we need x1 = 3 for negative 1 we need x2 = 3/2 then 1/3 + 2/3 = 1 same answer however I am not sure that it is accepted
@andryvokubadra2644
@andryvokubadra2644 Месяц назад
f(x²+1) = x⁴ + 2x² - 3 f(x) ? ================ f(x²+1) = x⁴ + 2x² - 3 f(x²+1) = (x⁴ + 2x² + 1) - 4 f(x²+1) = (x²+1)² - 4 f(x) = x² - 4 1:49
@JacekiElzbietaWojutynscy
@JacekiElzbietaWojutynscy Месяц назад
van Aubel's theorem. AF/FD= 8/7 + 6/7=2
@reachthestars98
@reachthestars98 Месяц назад
Yes, that also works but it is a more advanced theorem that not everyone knows about.
@체링-c7k
@체링-c7k 2 месяца назад
Why is the point M the circle of center?
@reachthestars98
@reachthestars98 Месяц назад
Point M was not part of the original question. First, we should find the center and radius of this circle. We can find the center by drawing the perpendicular bisectors of WZ and XY and finding their intersection point. This point happens to be the midpoint of AB, the hypotenuse, which I set as M. Hopefully this makes the problem more clear for you.
@체링-c7k
@체링-c7k Месяц назад
​@@reachthestars98Thanks 😊
@Nico-ni7hd
@Nico-ni7hd 2 месяца назад
why does the 5 cancel the log in base 5 ?
@MilesIsReal
@MilesIsReal 2 месяца назад
we know e^lnx = x. this is equivalent to e^log_e(x) = x. in general, for any base b, b^log_b(x) = x similarly, 5^log_5(x) = x
@Nico-ni7hd
@Nico-ni7hd 2 месяца назад
@@MilesIsReal ok thanks
@LearnoPhile-xy8om
@LearnoPhile-xy8om 2 месяца назад
Knew answer in 5 seconds
@pietergeerkens6324
@pietergeerkens6324 2 месяца назад
Nice! Took me nearly 8 seconds.
@7ymke
@7ymke 2 месяца назад
same for me
@ОаЬвлудаба
@ОаЬвлудаба 2 месяца назад
cool solution
@reachthestars98
@reachthestars98 Месяц назад
Thanks.
@Nomedecor
@Nomedecor 2 месяца назад
didn't understood a shit
@reachthestars98
@reachthestars98 2 месяца назад
If you didn’t understand the solution, feel free to ask me any questions about it.
@benyseus6325
@benyseus6325 2 месяца назад
It may not be obvious for some how you got from the x^2(y^2+1)+(y^2+1) step to (x^2+1)(y^2+1). You should also mention that the (y^2+1) multiplier was also factored out from both terms leaving (x^2+1). Just for clarity’s sake.
@reachthestars98
@reachthestars98 2 месяца назад
I will take note of that next time.
@MuhammadYusuf-ge2zy
@MuhammadYusuf-ge2zy 2 месяца назад
why does f(-x) the same as -f(x)? i want to learn but i cant grasp this part yet
@dylansalus9159
@dylansalus9159 2 месяца назад
This is a direct result of the definition of an odd function. Odd functions are defined as functions such that f(-x) = -f(x).
@MuhammadYusuf-ge2zy
@MuhammadYusuf-ge2zy 2 месяца назад
@@dylansalus9159 oh thanks bro🙏
@ronbannon
@ronbannon 2 месяца назад
The thumbnail does not match the video presentation. For those interested in the thumbnail's solution: g(x)=1/(1-x^2) and f(x)=x/(x^2-1)
@reachthestars98
@reachthestars98 2 месяца назад
Thanks for pointing that out. I fixed the thumbnail to match the video.
@ronbannon
@ronbannon 2 месяца назад
This function is periodic over the integers with a period of 2. You can start evaluating f(6) to move forward to see a period of 2, and you can also move backward with a little more effort. The assumption is that this function is defined over the integers.
@dainiusb1114
@dainiusb1114 2 месяца назад
Cos(EBC)=Sin(ABE). S=5^2-0.5*3*4-0.5*3*5*(3/5).
@divinesleeper
@divinesleeper 2 месяца назад
how do you know the period is 6?
@JossoJJossoJ
@JossoJJossoJ 2 месяца назад
The value??
@fhffhff
@fhffhff 2 месяца назад
BX=40/sin(60°-a)sina,s=20√3/sin (60°-a) FZ=41(√3-1)-40/sin(60°-a) sina a=-arccos((163-41√3)/2/√(83 24-41*82√3)+arccos(-40/√(8324- 41*82√3)) 11896 285/1487× s=20√3/sin120°=40
@fhffhff
@fhffhff 2 месяца назад
{y=X²-4X,0=(x-2)⁴-12(x-2)²-(x-2)+32; (z-2)³-17 3/4(z-2)-27 33/64=0 z=2+(880 1/2+√(1761²-71³*256/2 7)/2)¹/³/4+(..-..)..;2-(880 1/2+√ (1761²-71³*256/27)/2)¹/³/8-(..-..)..±√(3/4((880 1/2+√(1761²-71³*25 6/27)/2)¹/³/4+(..-..)..)²-17,75)i
@fhffhff
@fhffhff 2 месяца назад
x³(-√5x+x²+1)=0
@guilherminhookkk
@guilherminhookkk 2 месяца назад
you can find the inverse of the first one, f(x) = ax + b => f^-1(x) = (x - b)/a. Then equal to the other inverse, (x - b) / a = bx + a <=> x - b = abx + a^2, you find that: x = abx <=> ab = 1, and a^2 = -b. Furthermore, b = -a^2 and replacing in the other one: a(-a^2) = 1 <=> a = -1. Finally, (-1)^2 = - b, b = - 1, so a + b = -1 + (-1) = -2.
@fhffhff
@fhffhff 2 месяца назад
10/√(y²-100)(18-y) x=10√(y²-100)/y 5√(5+400/y²) 3/√(5+400/y²)=1,8√(y ²-100)/(18-y) 0=y⁴+45y³-450y²-18000 у=10,88 DF=16 324/536 x=19 12/17 K=653 (78 10/17)/134 ..=922(~✓)
@fhffhff
@fhffhff 2 месяца назад
10/√(y²-100)(18-y) x=10√(y²-100)/y 5√(5+400/y²) 3/√(5+400/y²)=1,8√(y ²-100)/(18-y) 0=y⁴+45y³-450y²-18000
@fhffhff
@fhffhff 2 месяца назад
x<log3(1/2) 0=3(3^x)²+2*3^x-1 3^x=(-2±√16)/6=1/3;-1>0 x=-1✓
@fhffhff
@fhffhff 2 месяца назад
a=10log5(2) b=10lg(2) 1/10log2(5)-1/10log2(10)=-1/10
@fhffhff
@fhffhff 2 месяца назад
S∆=√(12*3*4*5)=12√5 h=3√5 PQ=17/15√5
@fhffhff
@fhffhff 2 месяца назад
2log³10+6(log10-1)log²2=2 Ну в этом случае да.
@propofoldreams
@propofoldreams 2 месяца назад
just for fun. let O be center of circle. tan(OCB) = 1/2 thus tan(ECB) = 4/3 thus cos(DCE) = cos(90 - ECB) = sin(ECB) = 4/5 EC = 2 / cos(DCE) = 2 / (4/5) = 5/2
@niom-nx7kb
@niom-nx7kb 2 месяца назад
its crazy how the first step is thought of
@ki_li06
@ki_li06 2 месяца назад
I found another way to find the solution. Step 1) Find the inverse f^-1(x) of f(x) while assuming that the given f^-1(x) is different to the actual f^-1(x). Let's call the given f^-1(x) from now on g(x). Well, you'll get f^-1(x) = 1/a * x - b/a as the inverse of f(x) . Step 2) Equate the calculated f^-1(x) and g(x), which means that 1/a * x - b/a has to be equal to bx + a. Step 3) That leads to a system of equations with I) 1/a = b and II) -b/a = a, which is a solvable system of equation. You'll get a = -1 and b = -1. Step 4) Calculate a + b = -1 + (-1) = -2 (sry for my bad english)
@mathmachine4266
@mathmachine4266 2 месяца назад
b(ax+b)+a=x, abx+b²+a=x. ab=1, b²+a=0. a=1/b, b²+1/b=0. b isn't 0 or ∞, so b³+1=0. Meaning b is one of the 3 cube roots of -1. Meanwhile, a, being its reciprocal, is b's conjugate. Either a=b=-1, or a=(1±√(3)i)/2 and b=(1-±√(3)i)/2. Meaning either a+b=-2, or a+b=1. The two solutions are -2 and 1.
@RobMcelhenny
@RobMcelhenny 2 месяца назад
Easy ass shiiiiit bruh was this Olympiad considered hard or moderate??
@GrifGrey
@GrifGrey 2 месяца назад
it goes amc12 to aime to usamo to imo this is the easiest level of olympiad
@serhatkargn6333
@serhatkargn6333 2 месяца назад
I didn't got the last part, last row.
@ronbannon
@ronbannon 2 месяца назад
If a and b are not real, you will get three solutions. Sums would be -2, 1, 1.